Convergence or divergence of a series given divergent series












1












$begingroup$


If $displaystylesumlimits_{n=0}^{infty}a_n$ is divergent and $a_n > 0$ for all $n$, then does $displaystylesumlimits_{n=0}^{infty} dfrac{a_n}{1+n^2 a_n}$ converge or diverge?



The only progress I have is that if you consider the harmonic series, then we get the series with terms $dfrac{1}{n(n+1)}$, which converges.










share|cite|improve this question









$endgroup$

















    1












    $begingroup$


    If $displaystylesumlimits_{n=0}^{infty}a_n$ is divergent and $a_n > 0$ for all $n$, then does $displaystylesumlimits_{n=0}^{infty} dfrac{a_n}{1+n^2 a_n}$ converge or diverge?



    The only progress I have is that if you consider the harmonic series, then we get the series with terms $dfrac{1}{n(n+1)}$, which converges.










    share|cite|improve this question









    $endgroup$















      1












      1








      1


      0



      $begingroup$


      If $displaystylesumlimits_{n=0}^{infty}a_n$ is divergent and $a_n > 0$ for all $n$, then does $displaystylesumlimits_{n=0}^{infty} dfrac{a_n}{1+n^2 a_n}$ converge or diverge?



      The only progress I have is that if you consider the harmonic series, then we get the series with terms $dfrac{1}{n(n+1)}$, which converges.










      share|cite|improve this question









      $endgroup$




      If $displaystylesumlimits_{n=0}^{infty}a_n$ is divergent and $a_n > 0$ for all $n$, then does $displaystylesumlimits_{n=0}^{infty} dfrac{a_n}{1+n^2 a_n}$ converge or diverge?



      The only progress I have is that if you consider the harmonic series, then we get the series with terms $dfrac{1}{n(n+1)}$, which converges.







      sequences-and-series convergence






      share|cite|improve this question













      share|cite|improve this question











      share|cite|improve this question




      share|cite|improve this question










      asked 1 hour ago









      A. SmithA. Smith

      435




      435






















          1 Answer
          1






          active

          oldest

          votes


















          7












          $begingroup$

          HINT
          You have
          $$
          0<sum frac{a_n}{1+n^2a_n} < sum frac{a_n}{n^2a_n} = sum frac{1}{n^2}
          $$






          share|cite|improve this answer









          $endgroup$









          • 1




            $begingroup$
            (+1) Looks like the assumption that $sum a_n$ diverges is unnecessary.
            $endgroup$
            – Yanko
            1 hour ago










          • $begingroup$
            @Yanko likely professor tried to trick them into thinking this may diverge as well, not sure. Definitely not needed...
            $endgroup$
            – gt6989b
            1 hour ago











          Your Answer





          StackExchange.ifUsing("editor", function () {
          return StackExchange.using("mathjaxEditing", function () {
          StackExchange.MarkdownEditor.creationCallbacks.add(function (editor, postfix) {
          StackExchange.mathjaxEditing.prepareWmdForMathJax(editor, postfix, [["$", "$"], ["\\(","\\)"]]);
          });
          });
          }, "mathjax-editing");

          StackExchange.ready(function() {
          var channelOptions = {
          tags: "".split(" "),
          id: "69"
          };
          initTagRenderer("".split(" "), "".split(" "), channelOptions);

          StackExchange.using("externalEditor", function() {
          // Have to fire editor after snippets, if snippets enabled
          if (StackExchange.settings.snippets.snippetsEnabled) {
          StackExchange.using("snippets", function() {
          createEditor();
          });
          }
          else {
          createEditor();
          }
          });

          function createEditor() {
          StackExchange.prepareEditor({
          heartbeatType: 'answer',
          autoActivateHeartbeat: false,
          convertImagesToLinks: true,
          noModals: true,
          showLowRepImageUploadWarning: true,
          reputationToPostImages: 10,
          bindNavPrevention: true,
          postfix: "",
          imageUploader: {
          brandingHtml: "Powered by u003ca class="icon-imgur-white" href="https://imgur.com/"u003eu003c/au003e",
          contentPolicyHtml: "User contributions licensed under u003ca href="https://creativecommons.org/licenses/by-sa/3.0/"u003ecc by-sa 3.0 with attribution requiredu003c/au003e u003ca href="https://stackoverflow.com/legal/content-policy"u003e(content policy)u003c/au003e",
          allowUrls: true
          },
          noCode: true, onDemand: true,
          discardSelector: ".discard-answer"
          ,immediatelyShowMarkdownHelp:true
          });


          }
          });














          draft saved

          draft discarded


















          StackExchange.ready(
          function () {
          StackExchange.openid.initPostLogin('.new-post-login', 'https%3a%2f%2fmath.stackexchange.com%2fquestions%2f3077562%2fconvergence-or-divergence-of-a-series-given-divergent-series%23new-answer', 'question_page');
          }
          );

          Post as a guest















          Required, but never shown

























          1 Answer
          1






          active

          oldest

          votes








          1 Answer
          1






          active

          oldest

          votes









          active

          oldest

          votes






          active

          oldest

          votes









          7












          $begingroup$

          HINT
          You have
          $$
          0<sum frac{a_n}{1+n^2a_n} < sum frac{a_n}{n^2a_n} = sum frac{1}{n^2}
          $$






          share|cite|improve this answer









          $endgroup$









          • 1




            $begingroup$
            (+1) Looks like the assumption that $sum a_n$ diverges is unnecessary.
            $endgroup$
            – Yanko
            1 hour ago










          • $begingroup$
            @Yanko likely professor tried to trick them into thinking this may diverge as well, not sure. Definitely not needed...
            $endgroup$
            – gt6989b
            1 hour ago
















          7












          $begingroup$

          HINT
          You have
          $$
          0<sum frac{a_n}{1+n^2a_n} < sum frac{a_n}{n^2a_n} = sum frac{1}{n^2}
          $$






          share|cite|improve this answer









          $endgroup$









          • 1




            $begingroup$
            (+1) Looks like the assumption that $sum a_n$ diverges is unnecessary.
            $endgroup$
            – Yanko
            1 hour ago










          • $begingroup$
            @Yanko likely professor tried to trick them into thinking this may diverge as well, not sure. Definitely not needed...
            $endgroup$
            – gt6989b
            1 hour ago














          7












          7








          7





          $begingroup$

          HINT
          You have
          $$
          0<sum frac{a_n}{1+n^2a_n} < sum frac{a_n}{n^2a_n} = sum frac{1}{n^2}
          $$






          share|cite|improve this answer









          $endgroup$



          HINT
          You have
          $$
          0<sum frac{a_n}{1+n^2a_n} < sum frac{a_n}{n^2a_n} = sum frac{1}{n^2}
          $$







          share|cite|improve this answer












          share|cite|improve this answer



          share|cite|improve this answer










          answered 1 hour ago









          gt6989bgt6989b

          33.6k22453




          33.6k22453








          • 1




            $begingroup$
            (+1) Looks like the assumption that $sum a_n$ diverges is unnecessary.
            $endgroup$
            – Yanko
            1 hour ago










          • $begingroup$
            @Yanko likely professor tried to trick them into thinking this may diverge as well, not sure. Definitely not needed...
            $endgroup$
            – gt6989b
            1 hour ago














          • 1




            $begingroup$
            (+1) Looks like the assumption that $sum a_n$ diverges is unnecessary.
            $endgroup$
            – Yanko
            1 hour ago










          • $begingroup$
            @Yanko likely professor tried to trick them into thinking this may diverge as well, not sure. Definitely not needed...
            $endgroup$
            – gt6989b
            1 hour ago








          1




          1




          $begingroup$
          (+1) Looks like the assumption that $sum a_n$ diverges is unnecessary.
          $endgroup$
          – Yanko
          1 hour ago




          $begingroup$
          (+1) Looks like the assumption that $sum a_n$ diverges is unnecessary.
          $endgroup$
          – Yanko
          1 hour ago












          $begingroup$
          @Yanko likely professor tried to trick them into thinking this may diverge as well, not sure. Definitely not needed...
          $endgroup$
          – gt6989b
          1 hour ago




          $begingroup$
          @Yanko likely professor tried to trick them into thinking this may diverge as well, not sure. Definitely not needed...
          $endgroup$
          – gt6989b
          1 hour ago


















          draft saved

          draft discarded




















































          Thanks for contributing an answer to Mathematics Stack Exchange!


          • Please be sure to answer the question. Provide details and share your research!

          But avoid



          • Asking for help, clarification, or responding to other answers.

          • Making statements based on opinion; back them up with references or personal experience.


          Use MathJax to format equations. MathJax reference.


          To learn more, see our tips on writing great answers.




          draft saved


          draft discarded














          StackExchange.ready(
          function () {
          StackExchange.openid.initPostLogin('.new-post-login', 'https%3a%2f%2fmath.stackexchange.com%2fquestions%2f3077562%2fconvergence-or-divergence-of-a-series-given-divergent-series%23new-answer', 'question_page');
          }
          );

          Post as a guest















          Required, but never shown





















































          Required, but never shown














          Required, but never shown












          Required, but never shown







          Required, but never shown

































          Required, but never shown














          Required, but never shown












          Required, but never shown







          Required, but never shown







          Popular posts from this blog

          Polycentropodidae

          Magento 2 Error message: Invalid state change requested

          Paulmy